Definition of a Limit

Site: Saylor Academy
Course: MA005: Calculus I
Book: Definition of a Limit
Printed by: Guest user
Date: Friday, May 17, 2024, 4:16 AM

Description

Read this section to learn how a limit is defined. Work through practice problems 1-6.

Definition of Limit

It may seem strange that we have been using and calculating the values of limits for a while without having a precise definition of limit, but the history of mathematics shows that many concepts, including limits, were successfully used before they were precisely defined or even fully understood. We have chosen to follow the historical sequence in this chapter and to emphasize the intuitive and graphical meaning of limit because most students find these ideas and calculations easier than the definition. Also, this intuitive and graphical understanding of limit was sufficient for the first hundred years of the development of calculus (from Newton and Leibniz in the late 1600's to Cauchy in the early 1800's), and it is sufficient for using and understanding the results in beginning calculus. 

Mathematics, however, is more than a collection of useful tools, and part of its power and beauty comes from the fact that in mathematics terms are precisely defined and results are rigorously proved. Mathematical tastes (what is mathematically beautiful, interesting, useful) change over time, but because of these careful definitions and proofs, the results remain true, everywhere, and forever. Textbooks seldom give all of the definitions and proofs, but it is important to mathematics that such definitions and proofs exist. 

The goal of this section is to provide a precise definition of the limit of a function. The definition will not help you calculate the values of limits, but it provides a precise statement of what a limit is. The definition of limit is then used to verify the limits of some functions, and some general results are proved. 



Source: Dave Hoffman, https://s3.amazonaws.com/saylordotorg-resources/wwwresources/site/wp-content/uploads/2012/12/MA005-2.5-Definition-of-Limit.pdf
Creative Commons License This work is licensed under a Creative Commons Attribution 3.0 License.

The Intuitive Approach

The precise ("formal") definition of limit carefully defines the ideas that we have already been using graphically and intuitively. The following side–by–side columns show some of the phrases we have been using to describe limits, and those phrases, particularly the last ones, provide the basis to building the definition of limit. 

A Particular Limit

A General Limit

 \lim\limits_{x \rightarrow 3} 2 x-1=5  \lim\limits_{x \rightarrow a} f(x)=L
"as the values of x approach 3, the values of 2 \mathrm{x}-1 approach (are arbitrarily close to) 5 "as the values of x approach a, the values of \mathrm{f}(\mathrm{x}) approach (are arbitrarily close to) \mathrm{L}"
"when x is close to 3 (but not equal to 3 ), the value of 2 \mathrm{x}-1 is close to 5 "when \mathrm{x} is close to a (but not equal to a), the value of \mathrm{f}(\mathrm{x}) is close to \mathrm{L}
"we can guarantee that the values of \mathrm{f}(\mathrm{x})=2 \mathrm{x}-1 are as close to 5 as we want by starting with values of x sufficiently close to 3 (but not equal to 3 )"  "we can guarantee that the values of \mathrm{f}(\mathrm{x}) are as close to \mathrm{L} as we want by starting with values of x sufficiently close to a (but not equal to a)"  

Let's examine what the last phrase ("we can.."). means for the Particular Limit.


Example 1: We know \lim\limits_{\mathrm{x} \rightarrow 3} 2 \mathrm{x}-1=5. Show that we can guarantee that the values of \mathrm{f}(\mathrm{x})=2 \mathrm{x}-1 are as close to 5 as we want by starting with values of x sufficiently close to 3.

(a) What values of x guarantee that f(x)=2 x-1 is within \mathrm{1} unit of \mathrm{5}? (Fig. 1a)


Solution: "within \mathrm{1} unit of \mathrm{5}" means between 5-1=4 and 5+1=6, so the question can be rephrased as "for what values of x is y=2 x-1 between 4 and 6: 4  <  2 x-1  <  6? We want to know which values of x put the values of y=2 x-1 into the shaded band in Fig. 1a. The algebraic process is straightforward: solve 4  <  2 x-1  <  6 for x to get 5  <  2 x  <  7 and 2.5  <  x  <  3.5. We can restate this result as follows: "If \mathrm{x} is within \mathbf{0. 5} units of 3, then \mathrm{y}=2 \mathrm{x}-1 is within \mathrm{1} unit of \mathrm{5}". (Fig. 1b) 


Any smaller distance also satisfies the guarantee: e.g., "If x is within 0.4 units of 3, then y=2 x-1 is within 1 unit of 5". (Fig. 1c)


(b) What values of x guarantee the f(x)=2 x-1 is within 0.2 units of 5? (Fig. 2a)


Solution: "within 0.2 units of 5" means between 5-0.2=4.8 and 5+0.2=5.2, so the question can be rephrased as "for what values of x is y=2 x-1 between 4.8 and 5.2: 4.8  <  2 x-1  <  5.2?" Solving for x, we get 5.8  <  2 x   <  6.2 and 2.9  <  x  <  3.1. "If x is within 0.1 units of 3, then y=2 x-1 is within 0.2 units of 5." (Fig. 2b) Any smaller distance also satisfies the guarantee.


Rather than redoing these calculations for every possible distance from 5, we can do the work once, generally:

(c) What values of x guarantee that f(x)=2 x-1 is within E units of 5? (Fig. 3a)


Solution: "within \mathrm{E} unit of 5" means between 5-\mathrm{E} and 5+\mathrm{E}, so the question is "for what values of x is y=2 x-1 between 5-E and 5+\varepsilon: 5-\mathrm{E} < 2 \mathrm{x}-1 < 5+\mathrm{E}?" Solving 5-\mathrm{E}  <  2 \mathrm{x}-1  <  5+\mathrm{E} for \mathrm{x}, \mathrm{we} get 6-\mathrm{E} < 2 \mathrm{x}  <  6+\mathrm{E} and 3-\mathrm{E} / 2  <  \mathrm{x} < 3+\mathrm{E} / 2. "If \mathrm{x} is within \mathrm{E} / 2 units of 3, then y=2 x-1 is within E units of 5". (Fig. 3b) Any smaller distance also satisfies the guarantee.


Part (c) of Example 1 illustrates a little of the power of general solutions in mathematics. Rather than doing a new set of similar calculations every time someone demands that \mathrm{f}(\mathrm{x})=2 \mathrm{x}-1 be within some given distance of 5, we did the calculations once. And then we can respond for any given distance. For the question "What values of x guarantee that f(x)=2 x-1 is within 0.4,0.1 and 0.006 units of 5?", we can answer "If \mathrm{x} is within 0.2(=0.4 / 2), 0.05(=0.1 / 2) and 0.003(=0.006 / 2) units of 3".


Practice 1: \quad \lim\limits_{\mathrm{x} \rightarrow 2} 4 \mathrm{x}-5=3. What values of \mathrm{x} guarantee that \mathrm{f}(\mathrm{x})=4 \mathrm{x}-5 is within

(a) 1 unit of 3?

(b) 0.08 units of 3?

(c) E units of 3? (Fig. 4)


The same ideas work even if the graphs of the functions are not straight lines, but the calculations are more complicated.


Example 2: \quad \lim\limits_{\mathrm{x} \rightarrow 2} \mathrm{x}^{2}=4. (a) What values of \mathrm{x} guarantee that \mathrm{f}(\mathrm{x})=\mathrm{x}^{2} is within \mathrm{1} unit of \mathrm{4}? (b) Within 0.2 units of 4?

(Fig. 5a) State each answer in the form "If \mathrm{x} is within _____ units of \mathrm{2}, then \mathrm{f}(\mathrm{x}) is within \mathrm{1} (or 0.2) unit of \mathrm{4}".


Solution; (a) If x^{2} is within \mathrm{1} unit of \mathrm{4}, then 3 < x^{2} < 5 so \sqrt{3} < x < \sqrt{5}

or 1.732 < \mathrm{x} < 2.236. The interval containing these \mathrm{x} values extends from 2-\sqrt{3} \approx 0.268 units to the left of \mathrm{2} to \sqrt{5}-2 \approx 0.236 units to the right of 2. Since we want to specify a single distance on each side of \mathrm{2}, we can pick the smaller of the two distances, 0.236. (Fig. 5b)


"If x is within \mathrm{0.236} units of \mathrm{2}, then \mathrm{f}(\mathrm{x}) is within \mathrm{1} unit of \mathrm{4}".

(b) Similarly, if x^{2} is within 0.2 units of 4, then 3.8 < x^{2} < 4.2 so \sqrt{3.8} < \mathrm{x} < \sqrt{4.2} or 1.949 < \mathrm{x} < 2.049. The interval containing these x values extends from 2-\sqrt{3.8} \approx 0.051 units to the left of 2 to \sqrt{4.2}-2 \approx 0.049 units to the right of 2. Again picking the smaller of the two distances, "If x is within \mathrm{0.049} units of \mathrm{2}, then \mathrm{f}(\mathrm{x}) is within \mathrm{1} unit of \mathrm{4}".

The situation in Example 2 of different distances on the left and right sides is very common, and we always pick our single distance to be the smaller of the distances to the left and right. By using the smaller distance, we can be certain that if \mathrm{x} is within that smaller distance on either side, then the value of \mathrm{f}(\mathrm{x}) is within the specified distance of the value of the limit.

Practice 2: \quad \lim\limits_{x \rightarrow 9} \sqrt{x}=3. What values of x guarantee that \mathrm{f}(\mathrm{x})=\sqrt{\mathrm{x}} is within 1 unit of 3? Within 0.2 units of 3? (Fig. 6) State each answer in the form.


"If x is ______ within units of 2, then \mathrm{f}(\mathrm{x}) is within 1 (or 0.2) unit of 4".

The same ideas can also be used when the function and the specified distance are given graphically, and in that case we can give the answer graphically.


Example 3: In Fig. 7, \lim\limits_{\mathrm{x} \rightarrow 2} \mathrm{f}(\mathrm{x})=3. What values of \mathrm{x} guarantee that \mathrm{y}=\mathrm{f}(\mathrm{x}) is within \mathrm{E} units (given graphically) of 3? State your answer in the form "If \mathrm{x} is within _____ (show a distance D graphically) of 2, then \mathrm{f}(\mathrm{x}) is within \mathrm{E} units of 3".


Solution: \quad The solution process requires several steps as illustrated in Fig. 8:




i. Use the given distance \mathrm{E} to find the values 3-\mathrm{E} and 3+\mathrm{E} on the \mathrm{y}-axis.

ii. Sketch the horizontal band which has its lower edge at \mathrm{y}=3-\mathrm{E} and its upper edge at \mathrm{y}=3+\mathrm{E}.

iii. Find the first locations to the right and left of \mathrm{x}=2 where the graph of y=f(x) crosses the lines y=3-E and y=3+E and at these locations draw vertical lines to the \mathrm{x}-axis.

iv. On the \mathrm{x}-axis, graphically determine the distance from \mathrm{2} to the vertical line on the left (labeled \left.\mathrm{D}_{\mathrm{L}}\right) and from 2 to the vertical line on the right (labeled \left.\mathrm{D}_{\mathrm{R}}\right).

v. Let the length D be the smaller of the lengths D_{L} and D_{R}.


Practice 3: In Fig. 9, \lim\limits_{\mathrm{x} \rightarrow 3} \mathrm{f}(\mathrm{x})=1.8. What of x guarantee that y=f(x) is within E units of 1.8?


The Formal Definition of Limit

The ideas of the previous examples and practice problems can be stated for general functions and limits, and they provide the basis for the definition of limit which is given in the box. The use of the lower case Greek letters \varepsilon (epsilon) and \delta (delta) in the definition is standard, and this definition is sometimes called the "epsilon-delta" definition of limit.

Definition of \lim\limits_{\mathrm{x} \rightarrow \mathrm{a}} \mathbf{f}(\mathbf{x})=\mathbf{L}:

\lim\limits_{x \rightarrow a} f(x)=L means

for every given \varepsilon > 0 there is a \delta > 0 so that (Fig. 10)

if \quad x \quad is within \delta units of a( and x \neq a)
then f(x) is within \varepsilon units of L.
(Equivalently: |\mathrm{f}(\mathrm{x})-\mathrm{L}| < \varepsilon \quad whenever 0 < |\mathrm{x}-\mathrm{a}| < \delta).

In this definition, \varepsilon represents the given distance on either side of the limiting value \mathrm{y}=\mathrm{L}, and \delta is the distance on each side of the point x=a on the x-axis that we have been finding in the previous examples. This definition has the form of \mathrm{a} a "challenge and reponse:" for any positive challenge \varepsilon (make f(c) within \varepsilon of \mathrm{L}), there is a positive response \delta (start with \mathrm{x} within \delta of \mathrm{a} and x \neq a).

Example 4: In Fig. 11a, \lim\limits_{\mathrm{x} \rightarrow \mathrm{a}} \mathrm{f}(\mathrm{x})=\mathrm{L}, and a value for \varepsilon is given graphically as a length. Find a length for \delta that satisfies the definition of limit (so "if \mathrm{x} is within \delta of \mathrm{a}( and \mathrm{x} \neq \mathrm{a}), then \mathrm{f}(\mathrm{x}) is within \varepsilon of \mathrm{L}").


Solution: Follow the steps outlined in Example 3. The length for \delta is shown in Fig. 11b, and any shorter length for \delta also satisfies the definition.


Practice 4: In Fig. 12, \lim\limits_{\mathrm{x} \rightarrow \mathrm{a}} \mathrm{f}(\mathrm{x})=\mathrm{L}, and a value for \varepsilon is given graphically as a length. Find a length for \delta that satisfies the definition of limit.


Example 5: Prove that \lim\limits_{\mathrm{x} \rightarrow 3} 4 \mathrm{x}-5=7

Solution: We need to show that

"for every given \varepsilon > 0 there is a \delta > 0 so that

if \quad x \quad is within \delta units of 3( and x \neq 3)

then 4 x-5 is within \varepsilon units of 7".

Actually there are two things we need to do. First, we need to find a value for \delta (typically depending on \varepsilon), and, second, we need to show that our \delta really does satisfy the "if - then" part of the definition.

i. Finding \delta is similar to part (c) in Example 1 and Practice 1: assume 4 \mathrm{x}-5 is within \varepsilon units of 7 and solve for x. If 7-\varepsilon < 4 x-5 < 7+\varepsilon, then 12-\varepsilon < 4 x < 12+\varepsilon and 3-\varepsilon / 4 < x < 3+\varepsilon / 4, so x is within \varepsilon / 4 units of 3. Put \delta=\varepsilon / 4

ii. To show that \delta=\varepsilon / 4 satisfies the definition, we merely reverse the order of the steps in part i. Assume that x is within \delta units of 3. Then 3-\delta < x < 3+\delta so

\begin{array}{ll}3-\varepsilon / 4 < \mathrm{x} < 3+\varepsilon / 4 & (\text { replacing } \delta \text { with } \varepsilon / 4) \\ 12-\varepsilon < 4 \mathrm{x} < 12+\varepsilon & \text { (multiplying by 4), and } \\ 7-\varepsilon
 < 4 \mathrm{x}-5 < 7+\varepsilon & \text { (subtracting 5), so }\end{array} 

we can conclude that \mathrm{f}(\mathrm{x})=4 \mathrm{x}-5 is within \varepsilon units of 7. This formally verifies that \lim\limits_{\mathrm{x} \rightarrow 3} 4 \mathrm{x}-5=7.

Practice 5: Prove that \lim\limits_{x \rightarrow 4} 5 x+3=23.

The method used to prove the values of the limits for these particular linear functions can also be used to prove the following general result about the limits of linear functions.

Theorem: \quad \lim\limits_{\mathrm{x} \rightarrow \mathrm{a}} \mathrm{m} \mathrm{x}+\mathrm{b}=\mathrm{ma}+\mathrm{b}

Proof: Let f(x)=m x+b.

Case 1: \mathrm{m}=0. Then \mathrm{f}(\mathrm{x})=0 \mathrm{x}+\mathrm{b}=\mathrm{b} is simply a constant function, and any value for \delta > 0 satisfies the definition. Given any value of \varepsilon > 0, let \delta=1 (any positive value for \delta works). If x is is within 1 unit of \mathrm{a}, then f(x)-f(a)=b-b=0 < e, so we have shown that for any \varepsilon > 0, there is a \delta > 0 which satisfies the definition.

Case 2: \mathrm{m} \neq 0. Then f(x)=m x+b. For any \varepsilon > 0, put \delta=\frac{\varepsilon}{\mid m \mid} > 0. If x is within \delta=\frac{\varepsilon}{\mid m \mid} of \mathrm{a}, then

\mathrm{a}-\frac{\varepsilon}{\mid m \mid} < \mathrm{x} < \mathrm{a}+\frac{\varepsilon}{|\mathrm{m}|} \quad \text { so}-\frac{\varepsilon}{\mid m \mid} < \mathrm{x}-\mathrm{a} < \frac{\varepsilon}{\mid m \mid} \quad   and  |\mathrm{x}-\mathrm{a}| < \frac{\varepsilon}{\mid m \mid}

Then the distance between \mathrm{f}(\mathrm{x}) and \mathrm{L}=\mathrm{ma}+\mathrm{b} is |\mathrm{f}(\mathrm{x})-\mathrm{L}|=|(\mathrm{m} \mathrm{x}+\mathrm{b})-(\mathrm{ma}+\mathrm{b})|=\mid m|\cdot| \mathrm{x}-\mathrm{a}| < \mid m| \frac{\varepsilon}{\frac{\varepsilon}{\mathrm{m} \mid}}=\varepsilon so \mathrm{f}(\mathrm{x}) is within \varepsilon of \mathrm{L}=\mathrm{ma}+\mathrm{b}. (Fig. 13)


In each case, we have shown that "given any \varepsilon > 0, there is a \delta > 0" that satisfies the rest of the definition is satisfied.

If there is even a single value of \varepsilon for which there is no \delta, then the function does not satisfy the definition, and we say that the limit "does not exist".

Example 6: Let f(x)=\left\{\begin{array}{ll}2 & \text { if } x < 1 \\ 4 & \text { if } x > 1\end{array}\right. as is shown in Fig. 14.


Use the definition to prove that \lim\limits_{x \rightarrow 1} f(x) does not exist.

Solution: One common proof technique in mathematics is called "proof by contradiction," and that is the method we use here. Using that method in this case, (i) we assume that the limit does exist and equals some number \mathrm{L}, (ii) we show that this assumption leads to a contradiction, and (iii) we conclude that the assumption must have been false. Therefore, we conclude that the limit does not exist.

(i) Assume that the limit exists: \quad \lim\limits_{\mathrm{x} \rightarrow 1} \mathrm{f}(\mathrm{x})=\mathrm{L} for some value for \mathrm{L}. Let \varepsilon=\frac{1}{2}. (The definition says "for every ε" so we can pick this value. Why we chose this value for \varepsilon shows up later in the proof). Then, since we are assuming that the limit exists, there is a \delta > 0 so that if \mathrm{x} is within \delta of 1 then \mathrm{f}(\mathrm{x}) is within \varepsilon of \mathrm{L}.

(ii) Let x_{1} be between 1 and 1+\delta. Then x_{1} > 1 so f\left(x_{1}\right)=4. Also, x_{1} is within \delta of 1 so f\left(x_{1}\right)= 4 is within \frac{1}{2} of \mathrm{L}, and \mathrm{L} is between 3.5 and 4.5: 3.5 < \mathbf{L} < 4.5.

Let x_{2} be between 1 and 1-\delta. Then x_{2} < 1 so f\left(x_{2}\right)=2. Also, x_{2} is within \delta of 1 so f\left(x_{2}\right)= 2 is within \frac{1}{2} of \mathrm{L}, and \mathrm{L} is between 1.5 and 2.5: \mathbf{1. 5} < \mathbf{L} < \mathbf{2. 5}.

(iii) The two inequalities in bold print provide the contradiction we were hoping to find. There is no value \mathrm{L} that simultaneously satisfies 3.5 < \mathrm{L} < 4.5 and 1.5 < \mathrm{L} < 2.5, so we can conclude that our assumption was false and that f(x) does not have a limit as x \rightarrow 1.

Practice 6: Use the definition to prove that \lim\limits_{x \rightarrow 0} \frac{1}{x} does not exist (Fig. 15).



Two Limit Theorems

The theorems and their proofs are included here so you can see how such proofs proceed - you have already used these theorems to evaluate limits of functions. There are rigorous proofs of all of the other limit properties, but they are somewhat more complicated than the proofs given here.

Theorem: If \lim\limits_{\mathrm{x} \rightarrow \mathrm{a}} \mathrm{f}(\mathrm{x})=\mathrm{L}, then \lim\limits_{\mathrm{x} \rightarrow \mathrm{a}} \mathrm{k} \cdot \mathrm{f}(\mathrm{x})=\mathrm{k} \cdot \mathrm{L}.

Proof: Case \mathrm{k}=0: The Theorem is true but not very interesting: \quad \lim\limits_{\mathrm{x} \rightarrow \mathrm{a}} 0 \cdot \mathrm{f}(\mathrm{x})=\lim\limits_{\mathrm{x} \rightarrow \mathrm{a}} 0=0 \cdot \mathrm{L}.

Case k \neq 0: Since \lim\limits_{x \rightarrow a} f(x)=L, then, by the definition, for every \varepsilon > 0 there is a \delta > 0 so that |f(x)-L| < \varepsilon whenever |x-a| < \delta. For any \varepsilon > 0, we know \frac{\varepsilon}{|k|}
 > 0 and pick a value of \delta that satisfies |f(x)-L| < \frac{\varepsilon}{|k \mid} whenever |x-a| < \delta. When 

|x-a| < \delta \quad("x is within \delta of \mathrm{a}") then |f(x)-L| < \frac{\varepsilon}{|k|} \quad\left("f(x)\right. is within \frac{\varepsilon}{|k|} of L") so |k| \cdot|f(x)-L| < \varepsilon \quad (multiplying each side by |\mathrm{k}| > 0) and |\mathrm{k} \cdot \mathrm{f}(\mathrm{x})-\mathrm{k} \cdot \mathrm{L}| < \varepsilon \quad(\mathrm{k} \cdot \mathrm{f}(\mathrm{x}) is within \varepsilon of \mathrm{k} \cdot \mathrm{L})

Theorem: If \lim\limits_{x \rightarrow a} f(x)=L and \lim\limits_{x \rightarrow a} g(x)=M, then \lim\limits_{x \rightarrow a} f(x)+g(x)=L+M.

Proof: Assume that \lim\limits_{x \rightarrow a} f(x)=L and \lim\limits_{x \rightarrow a} g(x)=M. Then, given any \varepsilon > 0, we know \varepsilon / 2 > 0 and that there are deltas for \mathrm{f} and \mathrm{g}, \delta_{\mathrm{f}} and \delta_{\mathrm{g}}, so that 

if |x-a| < \delta_{f}, then |f(x)-L| < \varepsilon / 2 ("if x is within \delta_{f} of a, then f(x) is within \varepsilon / 2 of L", and 

if |x-a| < \delta_{g}, then \lg (x)-M \mid < \varepsilon / 2 ("if x is within \delta_{g} of \mathrm{a}, then g(x) is within \varepsilon / 2 of \mathrm{M}").

Let \delta be the smaller of \delta_{f} and \delta_{g}. If |x-a| < \delta, then |f(x)-L| < \varepsilon / 2 and |g(x)-M| < \varepsilon / 2 so

\mid(f(x)+g(x))-(L+M))|=|(f(x)-L)+(g(x)-M) \mid \quad (rearranging the terms)

 \leq |f(x)-L|+|g(x)-M| \quad (by the Triangle Inequality for absolute values)

< \frac{\varepsilon}{2}+\frac{\varepsilon}{2}=\varepsilon. \quad (by the definition of the limits for \mathrm{f} and \mathrm{g}).

Practice Problem Answers

Practice 1: (a) 3-1 < 4 x-5 < 3+1 so 7 < 4 x < 9 and 1.75 < x < 2.25: " x within 1 / 4 unit of \mathrm{2}".

(b) \quad 3-0.08 < 4 \mathrm{x}-5 < 3+0.08 so 7.92 < 4 \mathrm{x} < 8.08 and 1.98 < \mathrm{x} < 2.02: "\mathrm{x} within 0.02 units of \mathrm{2}".

(c) 3-\mathrm{E} < 4 \mathrm{x}-5 < 3+\mathrm{E} so 8_{-} \mathrm{E} < 4 \mathrm{x} < 8+\mathrm{E} and 2-\frac{\overline{\mathrm{E}}}{4} < \mathrm{x} < 2+{\mathrm{E}}{4}: "x within \mathrm{E} / 4 units of \mathrm{2}".


Practice 2: "within 1 unit of 3": If 2 < \sqrt{x} < 4, then 4 < x < 16 which extends from 5 units to the left of 9 to 7 units to right of 9. Using the smaller of these two distances from 9, "If x is within 5 units of 9, then \sqrt{x} is within \mathrm{1} unit of \mathrm{3}".

"within 0.2 units of 3": If 2.8 < \sqrt{x} < 3.2, then 7.84 < x < 10.24 which extends from 1.16 units to the left of 9 to 1.24 units to the right of 9. "If x is within 1.16 units of 9, then \sqrt{x} is wqithin 0.2 units of \mathrm{3}.


Practice 3: See Fig. 22.


Practice 4: See Fig. 23.


Practice 5: Given any \varepsilon>0, take \delta=\varepsilon / 5

If x is within \delta=\varepsilon / 5 of 4, then

4-\varepsilon / 5 < x < 4+\varepsilon / 5 so

\begin{array}{ll}-\varepsilon / 5 < \mathrm{x}-4 < \varepsilon / 5 & & \text { (subtracting 4) } \\ -\varepsilon < 5 \mathrm{x}-20 < \varepsilon & & \text { (multiplying by 5) } \\ -\varepsilon < (5 \mathrm{x}+3)-23 < \varepsilon & & \text { (rearranging to get the form we want)}\end{array}

We have shown that "for any \varepsilon > 0, there is a \delta > 0 (namely \delta=\varepsilon / 5)" so that the rest of the definition is satisfied.

Practice 6: This is a much more sophisticated (= harder) problem.

Using "proof by contradiction" as outlined in the solution to Example 6. 

(i) Assume that the limit exists: \lim _{\mathrm{x} \rightarrow 0} \frac{1}{x}=\mathrm{L} for some value for \mathrm{L}. Let \varepsilon=1. (The definition says "for every ε" so we can pick this value. For this limit, the definition fails for every \varepsilon > 0.) Then, since we are assuming that the limit exists, there is a \delta > 0 so that if x is within \delta of 0 then \mathrm{f}(\mathrm{x})=\frac{1}{\mathrm{x}} is within \varepsilon=1 of L.

(ii) (See Fig. 24) Let x_{1} be between \mathrm{0} and 0+\delta and also require that x_{1} < \frac{1}{2}. Then 0 < \mathrm{x}_{1} < \frac{1}{2} so \mathrm{f}\left(\mathrm{x}_{1}\right)=\frac{1}{\mathrm{x}_{1}} > 2. Since \mathrm{x}_{1} is within \delta of 0, \mathrm{f}\left(\mathrm{x}_{1}\right) > 2 is within \varepsilon=1 of \mathrm{L}, so L is greater than 2-\varepsilon=1: \mathbf{1} < \mathbf{L}.

Let \mathrm{x}_{2} be between 0 and 0-\delta and also require that \mathrm{x}_{2} > -\frac{1}{2}. Then 0 > \mathrm{x}_{2} > \frac{1}{2} so \mathrm{f}\left(\mathrm{x}_{2}\right)=\frac{1}{\mathrm{x}_{2}} < -2. Since \mathrm{x}_{2} is within \delta of 0, \mathrm{f}\left(\mathrm{x}_{2}\right) < -2 is

\varepsilon=1 of \mathrm{L}, so \mathrm{L} is less than -2+\varepsilon=-1:-\mathbf{1} > \mathbf{L}.

(iii) The two inequalities in bold print provide the contradiction we hoping to find, There is no value \mathrm{L} that satisfies

BOTH 1 < \mathbf{L} and \mathbf{L} < -\mathbf{1}

so we can conclude that our assumption was false and that \mathrm{f}(\mathrm{x})=\frac{1}{\mathrm{x}} \, does not have a limit as \mathrm{x} \rightarrow 0.